Đến nội dung

Nguyen Bao Khanh

Nguyen Bao Khanh

Đăng ký: 25-11-2022
Offline Đăng nhập: Hôm nay, 12:32
***--

#744499 $\left\{\begin{matrix} 3\sqrt[3]{y+(\sqrt{x^2+1...

Gửi bởi Nguyen Bao Khanh trong 05-04-2024 - 22:45

Dấu ngoặc ở pt thứ 2 đóng ở đâu ạ




#744486 $40$ số tự nhiên liên tiếp của tập $X$, trong đó có...

Gửi bởi Nguyen Bao Khanh trong 03-04-2024 - 23:13

Cho tập hợp $X=\{ 1;2;3;..;120\}$ gồm $120$ số nguyên dương đầu tiên, trong đó có $60$ số được viết bằng màu đỏ và $60$ số còn lại được viết bằng màu xanh. Chứng minh rằng tồn tại $40$ số tự nhiên liên tiếp của tập $X$, trong đó có $20$ số được viết bằng màu đỏ và $20$ số được viết bằng màu xanh.

(Thanh Hóa, chuyên Toán năm $23-24$)




#744413 $a^2 + \left\lceil {\frac{4a^2}{b}} \right\rcei...

Gửi bởi Nguyen Bao Khanh trong 27-03-2024 - 23:18

IMO Shortlist 2019 N8




#743678 Tổng 2 phần tử là số chính phương.

Gửi bởi Nguyen Bao Khanh trong 18-02-2024 - 10:39

Dạ em đã sửa ạ @perfectstrong @minhhaiproh




#743662 Tổng 2 phần tử là số chính phương.

Gửi bởi Nguyen Bao Khanh trong 18-02-2024 - 02:05

Giả sử tồn tại tập $B$ có $50$ phần tử bất kỳ của $S$ sao cho không tìm được 2 số có tổng là số chính phương.

  • $3 \in B$ thì do $3+78=81 =9^2$ nên $78 \not \in B \implies 22 \in B$. Lại có $22;3 \in B; 22+3=25=5^2$, mâu thuẫn
  • $97 \in B$ thì do $97+99=196=14^3$ nên $99 \not \in B \implies 1 \in B$ Lại có $1+8=3^2;1+48=7^2 \implies 48;8 \not \in B \implies 92;52 \in B$ nhưng $92+52=12^2$ nên mâu thuẫn.
  • $3;97\not\in B$ thì tập B phải chứa đúng 1 phần tử trong 48 cặp $(k;100-k);k\in\{1;…;49\mid k\neq3\}$ và 2 số $50;100$. Nhưng $50+(100-69)=9^2;100+69=13^2$ nên $69;100-69 \not \in B$. Khi đó B ít hơn 50 phần tử.

Do đó giả sử sai.




#743659 ​$(x+y\sqrt{5})^z=\sqrt{1+\sqrt{5...

Gửi bởi Nguyen Bao Khanh trong 18-02-2024 - 00:54

Bằng quy nạp, ta sẽ chứng minh $(x+y\sqrt5)^z=A+B\sqrt5$ với $A;B$ là số nguyên dương nào đó.

Thật vậy, trong trường hợp $z=1$ thì $A=x;B=y$.

Giả sử $(x+y\sqrt5)^i=A_i+B_i\sqrt5$ với mọi $i$ chạy từ $1$ đến $k$; $A_i;B_i$ là số nguyên dương.

Khi đó $(x+y\sqrt5)^{i+1}=(x+y\sqrt5)^i(x+y\sqrt5)=(A_i+B_i\sqrt5)(A_1+B_1\sqrt5)=(A_iA_1+5B_iB_1)+\sqrt5(B_iA_1+B_1A_i)$.

Do đó mệnh đề trên đúng với $i=k+1$. 

Theo nguyên lý quy nạp thì mệnh đề trên luôn đúng.

Khi đó $A+B\sqrt5=\sqrt{1+\sqrt5} \implies \sqrt5(2AB-1)=1-A^2-5B^2$. Nhận thấy $2AB-1 \neq 0$ nên chú ý $A;B$ nguyên dương, khi đó $\sqrt5 \in \mathbb{Q}$, vô lý.

Do đó phương trình vô nghiệm




#743644 $(x^3+y)(y^3+x)$ là lũy thừa của $2$.

Gửi bởi Nguyen Bao Khanh trong 17-02-2024 - 13:33

Hình như bạn đăng nhầm topic ?

Ta có $x;y;z \in \mathbb{N^*}; (x^3+y)(y^3+x)=2^z$.

Không mất tính tổng quát của bài toán, ta giả sử $x \ge y$.

Khi đó tồn tại số nguyên dương $m;n;~m \ge n$ sao cho $$ x^3+y=2^m;x+y^3=2^n \implies \left\{\begin{matrix} (x+y)(x^2-xy+y^2+1)=2^n(2^{m-n}+1) ~~~~(1)\\(x-y)(x^2+xy+y^2-1)=2^n(2^{m-n}-1)~~~~(2) \end{matrix}\right.$$ 

Ta nhận thấy $x;y$ cùng tính chẵn lẻ:

  1. $x;y$ chẵn. Khi đó $x^2+xy+y^2+1$ lẻ nhưng từ $(2)$, suy ra $(x-y)(x^2+xy+y^2-1) \vdots 2^n$ nên $(x-y) \vdots 2^n =x+y^3$. Khi đó $x=y$. Do đó $z$ là số chẵn; $z=2u; x^3+x=2^u \implies (x^2+1)x=2^u$ nhưng $x^2+1$ lẻ và lớn hơn 1 (loại).
  2. $x;y$ lẻ thì

+ $x=y$ thì $z$ là số chẵn; $z=2u; x^3+x=2^u \implies (x^2+1)x=2^u$ nhưng $x$ lẻ nên $x=1$. Khi đó $x=y=1;z=2$ (thỏa mãn).

+ $x>y$

Lại có $(x^2-xy+y^2+1)+(x^2+xy+y^2-1)=2(x^2+y^2) \equiv 4 \pmod 8$, nên trong 2 số có ít nhất 1 số không chia hết cho $8$. Khi đó $v_2(x^2-xy+y^2+1) \le 2$ hoặc $v_2(x^2+xy+y^2-1) \le 2$

  • $v_2(x^2-xy+y^2+1) \le 2$ thì $4(x+y) \vdots 2^n=x+y^3 \implies k \in \mathbb{N^*};~4(x+y)=k(x+y^3) \ge k(x+y) \implies k \in \{ 1;2;3;4\} $.

- $k=2$ thì $x=y^3-2y \implies 2^{n-1}=(y^2-1)y$ nhưng $y$ lẻ nên $y=1; x=-1$ (loại)

- $k=3$ thì $x=3y^3-4y \implies 2^{n-2}=y(y^2-1)$ nhưng $y$ lẻ nên $y=1;x=-1$ (loại)

- $k=4$ thì $y=1 \implies (x+1)(x^2-x+1)=2^m$ nhưng $x^2-x+1$ lẻ lớn hơn 1 (do $x>y=1$) (loại)

- $k=1$ thi $3x= y^3-4y \implies 3.2^{n-2}=y(y^2-1)$ nhưng $y$ lẻ nên $y \in \{ 1;3\} \implies (x;y)=(-1;1);(5;3) \implies (x;y;z)=(5;3;12)$ (thỏa mãn).

  •  $v_2(x^2+xy+y^2-1) \le 2$ thì $g \in \mathbb{N^*}; ~4(x-y)=g(x+y^3) > g(x-y) \implies g \in \{ 1;2;3\}$

- $g=1$ thì $3x=y^3+4y \implies 3.2^{n-2}=y(y^2+1)$ nhưng $y$ lẻ nên $y \in \{ 1;3\} \implies (x;y)=(5/3;1);(13;3) \implies z \in \varnothing$ (loại)

- $g=2$ thì $x=y^3+2y \implies 2^{n-1}=y(y^2+1)$ nhưng $y$ lẻ nên $y=1 \implies x=3 \implies z \in \varnothing$ (loại)

- $g=3$ thì $x=3y^3+4y \implies 2^{n-2}=y(y^2+1)$ nhưng $y$ lẻ nên $y=1;x=7 \implies z \in \varnothing$

Do đó $\boxed{(x;y)=(5;3);(1;1);(3;5)}$




#743635 Tìm số nguyên dương $k$ lớn nhất để $x^{3}-k(x+1)+1=...

Gửi bởi Nguyen Bao Khanh trong 16-02-2024 - 22:35

Khi đó theo định lý Viét cho phương trình bậc 3 thì $\begin{align} x_1+x_2+x_3=0 \\ x_1x_2+x_1x_3+x_3x_2=-k \\ x_1x_2x_3=1-k \end{align}$.
Vì phương trình chỉ có 2 nghiệm phân biệt nên không mất tính tổng quát thì $x_1=x_2 \neq x_3$.
Khi đó $\begin{cases} x_1^2+2x_1x_3=-k \\ x_3+2x_1=0 \end{cases} \implies \begin{cases} x_1^2+2x_1(-2x_1)=-k \\ x_3=-2x_1 \end{cases}$ $\implies \begin{cases} (x_1-2)^2=4-k \ge 0 \\ x_3=-2x_1 \end{cases}$. Khi đó $k \le 4 \implies k\in\{1;2;3;4\}$. Bằng thử trực tiếp, ta thấy $k \in \{3;1\}$ thì có 2 nghiệm phân biệt


#743615 HSG TP HN 2023-2024

Gửi bởi Nguyen Bao Khanh trong 16-02-2024 - 10:43

V1. Ta có $a;b;c \in \mathbb{N^*}; a^4+6a^2+2^b=c^2.$

Bỏ qua các trường hợp $b=1;b=2$.

Ta xét $b \ge 3$. 

  • Nếu $a$ lẻ thì $a^4+6a^2+2^b \equiv 1+6 \equiv 3 \pmod 4$, vô lý vì $c^2 \equiv 0;1 \pmod 4$ (loại)
  • Nếu $a$ chẵn thì khi đó ta đặt $a=2^k.t;k \in \mathbb{N^*}; t$ là số nguyên dương lẻ. Khi đó $2^{4k}.t^4+2^{2k+1}.3t^2+2^b=c^2$. Ta nhận thấy $4k > 2k+1$. Ta xét:
  1. Với $b>2k+1$ thì khi đó $2^{2k+1}(2^{2k-1}.t^4+3t^2+2^{b-2k-1})=c^2$. Vì vế phải là một số chính phương nên $c \vdots 2^{k+1}$. Khi đó $2^{2k-1}.t^4+3t^2+2^{b-2k-1} \vdots 2$. Khi đó $b=2k+1$, vô lý với $b >2k+1$.
  2. Với $b \le 2k+1 <4k$ thì khi đó $2^b(2^{4k-b}.t^4+3t^2.2^{2k+1-b}+1)=c^2$

+ $b$ lẻ thì $2^{4k-b}.t^4+3t^2.2^{2k+1-b}+1 \vdots 2 \implies 2k+1=b$. Khi đó $2^{2k}(2^{2k}.t^4+6t^2+2)=c^2$.

Khi đó tồn tại số nguyên dương $m$ sao cho $2^{2k}.t^4+6t^2+2=m^2 >(2^k.t^2)^2 \implies 6t^2 +1  \ge 2^{k+1}t^2 \implies k=1.$ Khi đó $4t^4+6t^2+2=m^2 \implies ...$

+ $b$ chẵn thì $b=2l; l \in \mathbb{N^*}$. Khi đó: $n \in \mathbb{N^*};n^2=(2^{2k-l}t^2)^2+6. t^2.2^{2k-2l}+1 \ge (2^{2k-l}t^2+1)^2 \implies 6t^22^{2k-2l} \ge 2.2^{2k-l}t^2 \implies  3.2^{-l-1} \ge 1$, vô lý vì $3.2^{-l-1}=\frac{3}{2^{l+1}} \le \frac{3}{4} <1$ 




#743574 Tìm $GTLN$ của tích $3$ số.

Gửi bởi Nguyen Bao Khanh trong 15-02-2024 - 12:49

Ta có $\sqrt{a}+\sqrt{b}=2023-\sqrt{c}\Rightarrow a+b-2\sqrt{ab}=2023^2-34\sqrt{7c}+c \Leftrightarrow T=2\sqrt{ab}-34\sqrt{7c}~~(T=a+b-c-2023^2 \in \mathbb{Z})\Rightarrow T^2+34^2.7c+68\sqrt{7c}=4ab\Rightarrow 68\sqrt{7c} \in \mathbb{Z} \implies \sqrt{7c} \in\mathbb{Q} \Longrightarrow \sqrt{7c} \in\mathbb{N}.$

Khi đó ta có thể đặt $c=7z^2$.Hoàn toàn tương tự $b=7y^2;a=7x^2;x;y;z\in \mathbb{N^*}$. Khi đó $x+y+z=17$.

Theo bất đẳng thức Cauchy: $xyz \le (\frac{x+y+z}{3})^3 \implies xyz \le 181$. Nhận thấy $181$ là số nguyên tố nên nếu $xyz=181$ thì có $1$ trong $3$ số $x;y;z$ bằng $181$, vô lý. Do vậy $xyz \ge 180$. Dấu "$=$" xảy ra chẳng hạn khi $x=6;y=6;z=5$.

Vậy $\text{max} \{ abc\}=7^3.6^4.5^2$, dấu đẳng thức xảy ra chẳng hạn khi $x=6;y=6;z=5$




#743547 Bài 4 - Cuộc thi giải toán "Mừng xuân Giáp Thìn, mừng VMF tròn 20 tuổi"

Gửi bởi Nguyen Bao Khanh trong 14-02-2024 - 12:32

Từ giả thiết, ta có $x^2-y^2+2xy=5$. 

Ta xét: $$3x^2+2y^2=x^2+4y^2+(2x^2-2y^2)=x^2-4xy+4y^2+2(x^2-y^2+2xy)=(x-2y)^2+2.5=(x-2y)^2+10$$.

Vì $(x-2y)^2 \ge 0; \forall x;y \in \mathbb{R}$ nên suy ra $$3x^2+2y^2=(x-2y)^2+10 \ge 10 ; \forall x;y \in \mathbb{R}.$$

Dấu $"="$ xảy ra khi và chỉ khi $\left\{\begin{matrix} x-2y=0\\x^2-y^2+2xy=5 \end{matrix}\right. \Leftrightarrow \left\{\begin{matrix} x=2y\\(2y)^2-y^2+2.2y.y=5 \end{matrix}\right.\Leftrightarrow \left\{\begin{matrix} x=2y\\ 7y^2=5 \end{matrix}\right. $ $\Leftrightarrow \left\{\begin{matrix} x=2y\\y=\pm \sqrt{\frac{5}{7}} \end{matrix}\right.$

Từ đó, ta có $(x;y) \in \{ (2\sqrt{\frac{5}{7}};\sqrt{\frac{5}{7}});(-2\sqrt{\frac{5}{7}};-\sqrt{\frac{5}{7}}) \}$ (thỏa mãn).

Vậy $\text{min} \{3x^2+2y^2\}=10$ khi và chỉ khi $(x;y) \in \{ (2\sqrt{\frac{5}{7}};\sqrt{\frac{5}{7}});(-2\sqrt{\frac{5}{7}};-\sqrt{\frac{5}{7}}) \}$

 

Nhận xét: Lời giải chính xác, ngắn gọn
Điểm: 10/10




#743530 Bài 3 - Cuộc thi giải toán "Mừng xuân Giáp Thìn, mừng VMF tròn 20 tuổi"

Gửi bởi Nguyen Bao Khanh trong 14-02-2024 - 00:16

1. Để thùng giấy ấy có thể đựng được $24$ lon nước đó sao cho chỉ có $1$ lớp được xếp thì chiều cao của thùng giấy ấy phải bằng chiều cao của $1$ lon nước hay chiều cao của thùng giấy đó là $14,5$ $cm$.

Hình vẽ dưới cho thấy mặt trên của thùng giấy sau khi xếp 24 lon Coca-cola vào thùng như giả thiết .

Khi đó chiều dài của thùng giấy đó phải bằng $6$ lần độ dài đường kính của $1$ lon nước Coca-cola hay chiều dài của thùng giấy đó là $6.5,5=33$ ($cm$).

Chiều rộng của thùng giấy đó phải bằng $4$ lần độ dài đường kính của $1$ lon nước Coca-cola hay

chiều rộng của thùng giấy đó là $4.5,5=22$ ($cm$).

Do đó số $m^2$ giấy ta cần bằng diện tích toàn phần của thùng giấy đó là: $(22+33).2.14,5+2.22.33 =3047$ ($cm^2$) $=0,3047$ ($m^2$).

Spoiler

421981456_402159725724958_87383513705026




#743479 Bài 2 - Cuộc thi giải toán "Mừng xuân Giáp Thìn, mừng VMF tròn 20 tuổi"

Gửi bởi Nguyen Bao Khanh trong 12-02-2024 - 15:05

 
  • Nếu $a \neq b $
Từ giả thiết, suy ra $ab(5a^2+5b^2-2) \vdots 5ab-1$ mà $GCD(5ab-1;ab)=GCD(1;ab)=1$ nên $5a^2+5b^2-2 \vdots 5ab-1$.
Khi đó chú ý $a;b \in \mathbb{N^*}$, ta đặt $$5a^2+5b^2-2=(5ab-1)k (k \in \mathbb{N^*})$$.
Giả sử $k$ là một số thỏa mãn bài toán. Cố định $k$. 
Xét tập hợp $$S=\{ (a;b); a;b \in \mathbb{N^*}; k=\frac{5a^2+5b^2-2}{5ab-1} \}$$.
Theo nguyên lý cực hạn, tồn tại $(a_0;b_0) \in S$, sao cho $a_0+b_0$ đạt giá trị nhỏ nhất. 
Do vai trò của $a_0$ và $b_0$ là như nhau nên không mất tính tổng quát, giả sử $a_0 > b_0$. 
Khi đó $$k=\frac{5a_0^2+5b_0^2-2}{5a_0b_0-1} \implies 5a_0^2+5b_0^2-2=5a_0b_0k-k \implies 5a_0^2-a_0.5b_0k+(5b_0^2+k-2)=0$$
Ta xét phương trình ẩn $X$, tham số $b_0;k$: $~~5X^2-X.5b_0k+(5b_0^2+k-2)=0$, có 1 nghiệm là $a_0$.
Do phương trình này là phương trình bậc 2 nên phương trình ấy còn 1 nghiệm nữa, ta gọi là $a_1$.
Theo hệ thức Vi-ét, ta có: $(*)\begin{cases} a_1+a_0=b_0k \\ a_0a_1=\frac{5b_0^2+k-2}{5}\end{cases}$
Chú ý $a_0;b_0;k \in \mathbb{N^*}$, từ phương trình thứ nhất suy ra $a_1 \in \mathbb{Z}$; từ phương trình thứ hai suy ra $a_1 > 0$.
Do đó $a_1 \in \mathbb{N^*}$. 
Do vậy $(a_1;b_0) \in S$. Từ cách chọn $(a_0;b_0)$, suy ra $a_1+b_0 \ge a_0+b_0 \implies a_1 \ge a_0.$ 
 
Từ $(*)$, suy ra $\frac{5b_0^2+k+3}{5}-b_0k=(a_0-1)(a_1-1) \ge (a_0-1)^2 > (b_0-1)^2$.
Do đó $\frac{5b_0^2+k+3}{5}-b_0k > (b_0-1)^2 \implies 5b_0k-10b_0+2-k <0 \implies (5b_0-1)(k-2) < 0$.
Do đó $k <2 \implies k=1.$
Khi đó $$5a_0^2-a_0.5b_0+(5b_0^2-1)=0 \implies 5(a_0^2-a_0b_0+b_0^2)=1$$
Vô lý vì vế trái chia hết cho $5$, còn vế phải thì không. (loại)
  • Nếu $a=b$ thì ta có điều cần chứng minh.
Do đó $a=b$ (đpcm)



#743461 $\frac{a^2}{b+c}+\frac{b^2}...

Gửi bởi Nguyen Bao Khanh trong 12-02-2024 - 00:19

Ta có

  •  $\sum_{a,b,c}^{}\frac{2a^2}{b+c}-(a+b+c)=\sum_{a,b,c}^{}\frac{(b-c)^2(a+b+c)}{(a+b)(a+c)}$
  • $\frac{3(a^3+b^3+c^3)}{a^2+b^2+c^2}-(a+b+c)=\frac{(a-b)^2(a+b)+(b-c)^2(b+c)+(a-c)^2(a+c)}{a^2+b^2+c^2}$

Ta chỉ cần chứng minh $\sum_{a,b,c}^{}\frac{(b-c)^2(a+b+c)}{(a+b)(a+c)} \ge \frac{(a-b)^2(a+b)+(b-c)^2(b+c)+(a-c)^2(a+c)}{a^2+b^2+c^2}$.

Ta chỉ cần chứng minh $\frac{(b-c)^2(a+b+c)}{(a+b)(a+c)} \ge \frac{(b-c)^2(b+c)}{c^2+a^2+b^2}$ nhưng $\frac{a+b+c}{(a+b)(a+c)} \ge \frac{a+b+c}{a^2+a^2+b^2+c^2}=\frac{b+c}{a^2+b^2+c^2}+\frac{a(a^2+b^2+c^2-ab-ac)}{(a^2+b^2+c^2)(2a^2+b^2+c^2)} \ge \frac{b+c}{a^2+b^2+c^2}$




#743457 Bài 1 - Cuộc thi giải toán "Mừng xuân Giáp Thìn, mừng VMF tròn 20 tuổi"

Gửi bởi Nguyen Bao Khanh trong 11-02-2024 - 23:35

Từ giả thiết, ta nhận xét rằng với $ \forall p; t \in \mathbb{N}; t$ lẻ thì ước số lẻ lớn nhất của $t$ là $t$ hay $a(t)=t$ và $a(2^p.t)=a(t)=t.$

Ta sẽ chứng minh đẳng thức sau: $\sum_{n=k}^{2k}a(n)=k^2+a(k); \forall k \in \mathbb{N^*} (*)$
Thật vậy, giả sử mệnh đề trên đúng đến $k=m$. Khi đó $\sum_{n=m}^{2m}a(n)=m^2+a(m); m \in \mathbb{N^*} $.
Khi đó kết hợp nhận xét trên, suy ra $\sum_{n=m+1}^{2(m+1)}a(n)=\sum_{n=m}^{2m}a(n)+a(2m+1)+a(2m+2)-a(m)=m^2+a(m)+(2m+1)+a(2(m+1))-a(m)=(m+1)^2+a(m+1).$
Từ đó, suy ra mệnh đề $(*)$ cũng đúng với $k=m+1$. Theo nguyên lý quy nạp suy ra đẳng thức $(*)$ đúng với $\forall k \in \mathbb{N^*}.$

Khi đó, ta thay $k=2024$, suy ra $\sum_{n=2024}^{4048}a(n)=2024^2+a(2024)=4096576+a(2^3.253)=4096576+253=4096829.$
Vậy giá trị của tổng trong đề bài là: 4096829.


Nhận xét: Định hướng đúng như đáp án. Lưu ý ngắt dòng nếu quá dài.
Điểm: 10/10